Naked Science Forum

Non Life Sciences => Physics, Astronomy & Cosmology => Topic started by: Bogie_smiles on 22/07/2018 11:10:57

Title: False Vacuum; Who, What, Where, When, Why?
Post by: Bogie_smiles on 22/07/2018 11:10:57
False Vacuum; Who, What, Where, When, Why?

In my last thread, “What is Nothingness?”, the phrase “false vacuum” was mentioned over 80 times. When it comes to the science of Cosmology, it seems that the concept of a false vacuum plays a role in different models and in different circumstances. What exactly is it and in what models of cosmology does it come into play? Does it always mean the same thing, or does it mean different things depending on the model?

Title: Re: False Vacuum; Who, What, Where, When, Why?
Post by: mrsmith2211 on 22/07/2018 21:06:30
"In quantum field theory, a false vacuum is a hypothetical vacuum that is somewhat, but not entirely, stable." I don't believe anything to be entirely stable, so is it just a time frame thing? https://en.wikipedia.org/wiki/False_vacuum
Title: Re: False Vacuum; Who, What, Where, When, Why?
Post by: Bogie_smiles on 23/07/2018 01:25:06
"In quantum field theory, a false vacuum is a hypothetical vacuum that is somewhat, but not entirely, stable." I don't believe anything to be entirely stable, so is it just a time frame thing? https://en.wikipedia.org/wiki/False_vacuum (https://en.wikipedia.org/wiki/False_vacuum)
It is an interesting Wiki, which I am still studying. One thing to notice is at the very top of the page, there are two tabs:
Article (https://en.wikipedia.org/wiki/False_vacuum)     Talk  https://en.wikipedia.org/wiki/Talk:False_vacuum (https://en.wikipedia.org/wiki/Talk:False_vacuum)


The Article is the Wiki page, and the Talk tab starts with an introduction to the WikiProject_Physics:

https://en.wikipedia.org/wiki/Wikipedia:WikiProject_Physics (https://en.wikipedia.org/wiki/Wikipedia:WikiProject_Physics)

And then it goes to the content of the talk page which has 21 statement headings.

It is going to be interesting to go over all of that in detail, thanks for participating mrsmith2211.
Title: Re: False Vacuum; Who, What, Where, When, Why?
Post by: jeffreyH on 23/07/2018 19:26:18
Investigate a question. How long will it take for a proton to decay. https://en.wikipedia.org/wiki/Proton_decay (https://en.wikipedia.org/wiki/Proton_decay) The proton may never decay or it may just be a very long time before it does. It is a question of stability. In the case of a false vacuum the stability arises from the idea that there is a 'hill' between the false and true vacuum potentials. In the same way there are barriers to proton decay. See especially:

"According to the Standard Model, protons, a type of baryon, are stable because baryon number (quark number) is conserved (under normal circumstances; see chiral anomaly for exception). Therefore, protons will not decay into other particles on their own, because they are the lightest (and therefore least energetic) baryon. Positron emission – a form of radioactive decay which sees a proton become a neutron – is not proton decay, since the proton interacts with other particles within the atom."
Title: Re: False Vacuum; Who, What, Where, When, Why?
Post by: jeffreyH on 23/07/2018 19:36:25
It may also be pertinent to read about the possibility of electron decay and just how likely that is. This also brings up the issue of violation of charge conservation. https://physics.aps.org/synopsis-for/10.1103/PhysRevLett.115.231802 (https://physics.aps.org/synopsis-for/10.1103/PhysRevLett.115.231802)
Title: Re: False Vacuum; Who, What, Where, When, Why?
Post by: Bill S on 23/07/2018 20:17:35
  The current understanding is that our vacuum has a positive energy equivalent to the mass of three hydrogen atoms per cubic metre, not a lot, but enough to have an enormous impact. 

    In theory, a false vacuum is a higher-energy version, and is characteristically unstable.  Typically, it decays into true vacuum in a small fraction of a second, and its energy is released in a “fireball” of elementary particles.   
 
    In trying to establish a non-technical image of the vacuum, and its energy, the best analogy I found was that of an irregular surface of waves and troughs representing this energy, and considered as a “landscape”.  The bottom of the lowest valley is considered to be the “true vacuum”, that in which our Universe exists.  Crests of waves, and the slopes linking them to the valleys, represent “false “vacuum” states in which the vacuum energy is higher.  Essentially, these are very unstable states. 
Title: Re: False Vacuum; Who, What, Where, When, Why?
Post by: jeffreyH on 23/07/2018 21:45:26
I'm impressed Bill.
Title: Re: False Vacuum; Who, What, Where, When, Why?
Post by: Bill S on 24/07/2018 00:09:48
Thanks, Jeffrey.  I guess that means I got somewhere near the "authorised version".
Title: Re: False Vacuum; Who, What, Where, When, Why?
Post by: Colin2B on 24/07/2018 09:21:25
I like your analogy @Bill S
I would view the higher valleys as tarns - lakes cut off from lower areas. Interesting that these areas are considered to dissipate by quantum tunneling which is why they are unstable.
Title: Re: False Vacuum; Who, What, Where, When, Why?
Post by: Bill S on 24/07/2018 20:46:41
Quote from: Colin
I would view the higher valleys as tarns - lakes cut off from lower areas.


So far, so good.  Considering a landscape in which a false vacuum state is represented by a valley at an elevation higher than the true vacuum; this would provide the necessary stability. 

What is it that is stable in these valleys?
This necessitates looking at the “inhabitants” of the landscape; often depicted in Pop Sci books as little spheres that can roll about on the hilly landscape.  The best identification of these spheres I have found is that they are scalar fields.  Their energy is dictated by the vertical position they occupy, at any given instant, on the landscape. 

How big are these scalar fields? 
This is a question that is commonly ignored, possibly with good reason, but in developing his argument about the origin of the universe, Vilenkin treats them as quantum objects.  Presumably, this is how we should see them in order to progress to your comment on quantum tunnelling.   
Title: Re: False Vacuum; Who, What, Where, When, Why?
Post by: Bill S on 24/07/2018 23:50:06
I’ve found some things that seem to conflict, (surprise!).

One image is that of a scalar field resting with stability in the lowest valley.

Contrast this with statements like: “the bottom of the valley into which the sphere rolls is considered to be the true vacuum, and this is where the scalar field discharges its energy.”

There seems to be a dichotomy between resting in the bottom of the valley, and that “fireball” of elementary particles, mentioned earlier.

The most sensible way round this, that I could see, was that the unstable quon came to rest in the bottom of the valley, discharged its energy and became a stable, expanding, universe.   Does that seem reasonable?
Title: Re: False Vacuum; Who, What, Where, When, Why?
Post by: Bogie_smiles on 25/07/2018 05:07:14
Reply #11


Investigate a question. How long will it take for a proton to decay. https://en.wikipedia.org/wiki/Proton_decay (https://en.wikipedia.org/wiki/Proton_decay) The proton may never decay or it may just be a very long time before it does. It is a question of stability. In the case of a false vacuum the stability arises from the idea that there is a 'hill' between the false and true vacuum potentials. In the same way there are barriers to proton decay. See especially:

"According to the Standard Model, protons, a type of baryon, are stable because baryon number (quark number) is conserved (under normal circumstances; see chiral anomaly for exception). Therefore, protons will not decay into other particles on their own, because they are the lightest (and therefore least energetic) baryon. Positron emission – a form of radioactive decay which sees a proton become a neutron – is not proton decay, since the proton interacts with other particles within the atom."


It may also be pertinent to read about the possibility of electron decay and just how likely that is. This also brings up the issue of violation of charge conservation. https://physics.aps.org/synopsis-for/10.1103/PhysRevLett.115.231802 (https://physics.aps.org/synopsis-for/10.1103/PhysRevLett.115.231802)


Let’s say, for purposes of this thread anyway, at least for now, that when the stable fundamental particles formed after the Big Bang, that the protons and electrons were so stable that there is no real evidence of any meaningful decay beyond their formation.

Title: Re: False Vacuum; Who, What, Where, When, Why?
Post by: Bogie_smiles on 25/07/2018 05:10:52
Reply #12

  The current understanding is that our vacuum has a positive energy equivalent to the mass of three hydrogen atoms per cubic metre, not a lot, but enough to have an enormous impact. 

    In theory, a false vacuum is a higher-energy version, and is characteristically unstable.  Typically, it decays into true vacuum in a small fraction of a second, and its energy is released in a “fireball” of elementary particles.   
 
    In trying to establish a non-technical image of the vacuum, and its energy, the best analogy I found was that of an irregular surface of waves and troughs representing this energy, and considered as a “landscape”.  The bottom of the lowest valley is considered to be the “true vacuum”, that in which our Universe exists.  Crests of waves, and the slopes linking them to the valleys, represent “false “vacuum” states in which the vacuum energy is higher.  Essentially, these are very unstable states. 

I like your analogy @Bill S
I would view the higher valleys as tarns - lakes cut off from lower areas. Interesting that these areas are considered to dissipate by quantum tunneling which is why they are unstable.
Quote from: Colin
I would view the higher valleys as tarns - lakes cut off from lower areas.


So far, so good.  Considering a landscape in which a false vacuum state is represented by a valley at an elevation higher than the true vacuum; this would provide the necessary stability. 

What is it that is stable in these valleys?
This necessitates looking at the “inhabitants” of the landscape; often depicted in Pop Sci books as little spheres that can roll about on the hilly landscape.  The best identification of these spheres I have found is that they are scalar fields.  Their energy is dictated by the vertical position they occupy, at any given instant, on the landscape. 

How big are these scalar fields? 
This is a question that is commonly ignored, possibly with good reason, but in developing his argument about the origin of the universe, Vilenkin treats them as quantum objects.  Presumably, this is how we should see them in order to progress to your comment on quantum tunnelling.   

I’ve found some things that seem to conflict, (surprise!).

One image is that of a scalar field resting with stability in the lowest valley.

Contrast this with statements like: “the bottom of the valley into which the sphere rolls is considered to be the true vacuum, and this is where the scalar field discharges its energy.”

There seems to be a dichotomy between resting in the bottom of the valley, and that “fireball” of elementary particles, mentioned earlier.

The most sensible way round this, that I could see, was that the unstable quon came to rest in the bottom of the valley, discharged its energy and became a stable, expanding, universe.   Does that seem reasonable?



Reply #12

It does seem reasonable to consider it that way.

I quite agree with you about the comment in Reply #5, where you posted:
  …

    In theory, a false vacuum is a higher-energy version, and is characteristically unstable.  Typically, it decays into true vacuum in a small fraction of a second, and its energy is released in a “fireball” of elementary particles.   
Let’s not move forward too quickly; my layman enthusiasm needs time to equate the analysis of the false vacuum theories to what we observe and theorize about our known universe and the observable portion of it.

The fireball comment can be equated to the big bang event that standard theory implies occurred at t=0, followed in an instant by rapid inflation, and eventual decay of that hot dense ball of energy into the extremely stable particles that jeffreyH pointed out.

If I may, I would take the fireball (big bang), and the decay of the extreme density of the fireball into a series of exotic particles until the protons (quarks), neutrons, and electrons became stable particles, and formed atoms, and began emitting photons. That would put us at about 300 thousand years into our expanding big bang observable/known universe. It is there that I would like to reason out something with you.

If you look at the link provided by mrsmith2211, under the “expansion of bubble” heading, you see a point in the Wiki on “false vacuum” that says:
“If two bubbles are nucleated and they eventually collide, it is thought that particle production would occur where the walls collide.

Can that be the point that Bill S mentions as the “fireball”, which I think logically leads to the decay and eventual formation of the stable particles, then atoms, and the surface of last scattering where the new atoms began to emit photons (light)?
Title: Re: False Vacuum; Who, What, Where, When, Why?
Post by: Bill S on 25/07/2018 11:06:30
Quote from: Bogie_Smiles
“If two bubbles are nucleated and they eventually collide, it is thought that particle production would occur where the walls collide.

Can that be the point that Bill S mentions as the “fireball”,

You're way ahead of me there.  So far I've reached the point of a single "sphere" dropping to a lower energy level.  At first sight, there seems to be no problem with that if the sphere is just rolling about on the landscape. 

Enter the nit-picker!  Where did the sphere come from?  If it can roll only downhill, how did it get to the higher level in the first place?

We still haven't reached the tunnelling that Colin mentioned; but one step at a time.
Title: Re: False Vacuum; Who, What, Where, When, Why?
Post by: Bill S on 25/07/2018 11:34:03
Quote
If you look at the link provided by mrsmith2211

I meant to acknowledge that link.  It has a lot of interesting stuff in it, thanks mrsmith, I'm not there yet, but stay tuned.
Title: Re: False Vacuum; Who, What, Where, When, Why?
Post by: Bogie_smiles on 25/07/2018 11:45:00
Let's go back then; where did you get the "fireball of elementary particles" quote? in paragraph 2 of this post?
  The current understanding is that our vacuum has a positive energy equivalent to the mass of three hydrogen atoms per cubic metre, not a lot, but enough to have an enormous impact. 

    In theory, a false vacuum is a higher-energy version, and is characteristically unstable.  Typically, it decays into true vacuum in a small fraction of a second, and its energy is released in a “fireball” of elementary particles.   
 
    In trying to establish a non-technical image of the vacuum, and its energy, the best analogy I found was that of an irregular surface of waves and troughs representing this energy, and considered as a “landscape”.  The bottom of the lowest valley is considered to be the “true vacuum”, that in which our Universe exists.  Crests of waves, and the slopes linking them to the valleys, represent “false “vacuum” states in which the vacuum energy is higher.  Essentially, these are very unstable states. 

Title: Re: False Vacuum; Who, What, Where, When, Why?
Post by: Bill S on 25/07/2018 11:58:51
Quote
...where did you get the "fireball of elementary particles" quote?

Alex Vilenkin, "Many Worlds in One".
Title: Re: False Vacuum; Who, What, Where, When, Why?
Post by: Bogie_smiles on 25/07/2018 13:38:55
Quote
...where did you get the "fireball of elementary particles" quote?

Alex Vilenkin, "Many Worlds in One".
Then is it a reference to the Big Bang event and particle formation, isn't it?
Title: Re: False Vacuum; Who, What, Where, When, Why?
Post by: Bill S on 25/07/2018 13:43:15
It's part of his "universe from nothing" idea.  I'd be happy to look more closely at that later, but at the moment I'm sort of plodding.
Title: Re: False Vacuum; Who, What, Where, When, Why?
Post by: Bill S on 25/07/2018 13:47:56
Questions are accumulating.  I know it makes sense to seek answers for just a few at a time, but I have to grab opportunities to post, so here are some more, before they go out of my head.

We’ve considered that some stability may be achieved by a false vacuum state, and this would be represented on the vacuum landscape by a valley at a higher level than the true vacuum. 

What might cause this higher level valley to form?
Is it formed because a scalar field has, somehow, attained stability and thus distorts the landscape, or is it the presence of the “valley” that imparts stability to the field?

Having formed, this scenario gives some stability to the false vacuum state, because a field would need an input of appropriately directed energy to enable it to climb out of the valley.   

When a field “rolled” into a valley, why would it remain in its original state? 
How would it “know” that this valley was not the minimum energy state? 
How likely is it that a new universe would be created at this false vacuum level?
Would I be right in thinking that there is a view held that our Universe could be at one of these false vacuum levels?
Title: Re: False Vacuum; Who, What, Where, When, Why?
Post by: Bogie_smiles on 25/07/2018 16:05:15
Thank you for grabbing those opportunities to post, and for presenting those questions to the thread. I hope everyone who understands the theories involved will contribute what they can.
Title: Re: False Vacuum; Who, What, Where, When, Why?
Post by: Bill S on 25/07/2018 19:14:35
Quote
  I hope everyone who understands the theories involved will contribute what they can.

That could be wishful thinking.  This is an excellent forum and we “hitch-hikers” are fortunate to have such good help and support.  However, the experts/professionals who provide that are probably very busy people, and should be applauded for their generosity. (How many answers does that earn me? 😊) 

I am aware of the fact that I am flooding this thread with questions and may continue to do that,so I’ll be grateful for any that are answered. 

In the meantime, Bogie_Smiles, we can kick ideas around and hope others will join in.
Title: Re: False Vacuum; Who, What, Where, When, Why?
Post by: Bill S on 25/07/2018 19:47:52
Let’s consider, now, a landscape in which there is a valley at a higher energy level than the minimum. This is where the scalar field comes to rest. We have not established any reason why it might, or might not, discharge its energy, but since tunnelling is going to be considered, it must be that the field remains as a quon.

QM tells us that the energy input needed to lift the sphere over the energy barrier can be eliminated if the sphere tunnels through the barrier.  Such an occurrence would be vanishingly unlikely if the sphere were anything other than a quon; but a quon is what we are visualising.

This brings us back to “scale”. Do we have to imagine the valley and the sphere as being sub-microscopic?

Is the energy barrier quantum scale?
Does it have any physical extent at all?

Vilenkin says: “Despite the similarity between the tunneling of a ball and that of a scalar field, there is an important difference.  The ball tunnels between two different points in space, while for the scalar field the tunneling is between two different values of the field at the same location.”

I think I need either help visualising this, or a good single malt – large!
Title: Re: False Vacuum; Who, What, Where, When, Why?
Post by: Bogie_smiles on 25/07/2018 20:03:37

That could be wishful thinking.  This is an excellent forum and we “hitch-hikers” are fortunate to have such good help and support.  However, the experts/professionals who provide that are probably very busy people, and should be applauded for their generosity. (How many answers does that earn me? 😊


I am aware of the fact that I am flooding this thread with questions and may continue to do that,so I’ll be grateful for any that are answered. 

In the meantime, Bogie_Smiles, we can kick ideas around and hope others will join in.

Let’s consider, now, a landscape in which there is a valley at a higher energy level than the minimum. This is where the scalar field comes to rest. We have not established any reason why it might, or might not, discharge its energy, but since tunnelling is going to be considered, it must be that the field remains as a quon.

QM tells us that the energy input needed to lift the sphere over the energy barrier can be eliminated if the sphere tunnels through the barrier.  Such an occurrence would be vanishingly unlikely if the sphere were anything other than a quon; but a quon is what we are visualising.

This brings us back to “scale”. Do we have to imagine the valley and the sphere as being sub-microscopic?

Is the energy barrier quantum scale?
Does it have any physical extent at all?

Vilenkin says: “Despite the similarity between the tunneling of a ball and that of a scalar field, there is an important difference.  The ball tunnels between two different points in space, while for the scalar field the tunneling is between two different values of the field at the same location.”

I think I need either help visualising this, or a good single malt – large!


That is a generous offer that we kick some ideas around, and probably a good way to proceed. Before we attempt to get into the landscape  and tunneling related to the Wiki about the false and true vacuum, earlier I lamented the fact that it could be equated to what we observe of our known universe.

To state my view of what we see: we see a vast observable patch of space filled with galactic structure, and there is a shift toward the red in the spectrum of light coming from those galaxies that is convincing evidence that the components of that structure are moving away from us and each other, and the rate of that separation appears to be accelerating. Also, we observe a cosmic microwave background (CMB), and within that background there are two anomalies, the wide angle temperature difference, called hemispherical asymmetry or dipole anisotropy (http://iopscience.iop.org/article/10.1086/518091/meta (http://iopscience.iop.org/article/10.1086/518091/meta)), and there is a large cold spot (https://en.wikipedia.org/wiki/CMB_cold_spot (https://en.wikipedia.org/wiki/CMB_cold_spot)) in the temperature readings from the WMAP and Planck surveys. Is that a summary on the observable universe that you agree with?


Do you agree with me that one objective is to put what we observe of our known universe into the context of the false vs the true vacuum state as defined by QFT?


Do you agree with jeffreyH, as I do, that what we can observe is composed of some very stable particles that we suppose to have existed for about 14 billion years or so, and that should be able to exist in our particular vacuum state for a very long time into the future, whether or not our state is consistent with a false vacuum of a true vacuum as defined in QFT?

You can go to the NakedScientists image gallery and get this image link if you want to include it as an aid to the discussion at any time.https://www.thenakedscientists.com/forum/gallery/43933_25_07_18_6_47_50.jpeg (https://www.thenakedscientists.com/forum/gallery/43933_25_07_18_6_47_50.jpeg)(https://www.thenakedscientists.com/forum/gallery/43933_25_07_18_6_47_50.jpeg)
Title: Re: False Vacuum; Who, What, Where, When, Why?
Post by: Bill S on 26/07/2018 12:57:06
Quote
To state my view of what we see: we see a vast observable patch of space filled with galactic structure, and there is a shift toward the red in the spectrum of light coming from those galaxies that is convincing evidence that the components of that structure are moving away from us and each other, and the rate of that separation appears to be accelerating. Also, we observe a cosmic microwave background (CMB), and within that background there are two anomalies, the wide angle temperature difference, called hemispherical asymmetry or dipole anisotropy (http://iopscience.iop.org/article/10.1086/518091/meta), and there is a large cold spot (https://en.wikipedia.org/wiki/CMB_cold_spot) in the temperature readings from the WMAP and Planck surveys. Is that a summary on the observable universe that you agree with?

I don’t know much about asymmetries in the CMB, but, that aside, I’m good with your view. 
I’m not clear as to how you link this to the false vacuum of the OP.


Quote
Do you agree with me that one objective is to put what we observe of our known universe into the context of the false vs the true vacuum state as defined by QFT?

Keeping in mind that the false vacuum is hypothetical, I would say that any apparent link between it and observable phenomena would be worth investigating.

Quote
Do you agree with jeffreyH, as I do, that what we can observe is composed of some very stable particles that we suppose to have existed for about 14 billion years or so, and that should be able to exist in our particular vacuum state for a very long time into the future, whether or not our state is consistent with a false vacuum of a true vacuum as defined in QFT?

Yes.  Would I dare disagree with Jeffrey? 😊

Good image. Brings to mind the limerick about the young girl from Devizes.
Title: Re: False Vacuum; Who, What, Where, When, Why?
Post by: Bogie_smiles on 26/07/2018 13:56:05

I don’t know much about asymmetries in the CMB, but, that aside, I’m good with your view. 
I’m not clear as to how you link this to the false vacuum of the OP.


Keeping in mind that the false vacuum is hypothetical, I would say that any apparent link between it and observable phenomena would be worth investigating.


Yes.  Would I dare disagree with Jeffrey? 😊

Good image. Brings to mind the limerick about the young girl from Devizes.

There are links in my post to the CMB anomalies which I have been looking at, and the longer scientists examine the data and seek explanations, the more it is becoming important in regard to the nature of the energy density background we have experienced since the initial expansion, and that we exist in today.

It is that energy density, and its directional variance in the scalar patch of space that we can observe, that links it to the hypotheticals about false and true vacuums. The density of the vacuum during the period from the decay of the hot dense ball of energy, expanding and cooling to the point of exotic particles, Higgs fields, and an eventual expansion and cooling to reach background density that supports the extremely stable particles.

Talking about the various designations of the vacuum in the Wiki posted by @mrsmith2211, https://en.wikipedia.org/wiki/False_vacuum (https://en.wikipedia.org/wiki/False_vacuum):
In QFT there is the false vacuum that is somewhat stable, and can evolve over a very long period of time to become more stable, as our observable universe seems to have done. We could have moved from a state that equates to a local minimum as the local vacuum density declines, to a more stable state.

My take on the Wiki is that sequence of events foretells of a more stable vacuum of lower density surrounding our “bubble”. Do you get the same take from the first few sections of the Wiki about vacuums and the stability of vacuums?

And about your reference to a young girl from Devizes; I’m familiar with one about a young man, but I don’t think the content is within the acceptable NS guidelines, so we better move on.
Title: Re: False Vacuum; Who, What, Where, When, Why?
Post by: Bogie_smiles on 26/07/2018 14:51:22
This is a movie that looks a lot like what I would envision as the type of random energy density fluctuations that could lead to the formation of a single bubble nucleating in a patch of true vacuum in QFT. It is a simulation within a cubic volume, and it would look better if it was centered, but you can get the idea. It is a link I found searching around the Internet:
http://apollon.issp.u-tokyo.ac.jp/~watanabe/sample/movie/nucleation_th02.mpg (http://apollon.issp.u-tokyo.ac.jp/~watanabe/sample/movie/nucleation_th02.mpg)


I don't mean for this to be anything more than a hypothetical visual, and I don't think it represents the process from which our observable universe emerged, but for talking purposes I think it is a useful visual.
Title: Re: False Vacuum; Who, What, Where, When, Why?
Post by: Bill S on 26/07/2018 15:30:08
Quote
There are links in my post to the CMB anomalies which I have been looking at

As usual, I'm suffering from lack of time.  Much as I would like to follow links and read lots of peripheral things, keeping up with posts is about the limit.

I was going to post a link to the limerick, but the poetry was so bad I didn't want to be associated with it.  This is a bit better:

There was a young girl from Devizes
Whose breasts were of different sizes,
One was quite small,
Really nothing at all,
But the other was big and won prizes.

Quick!  Read it before the Mods take it down. :)
Title: Re: False Vacuum; Who, What, Where, When, Why?
Post by: Bogie_smiles on 26/07/2018 15:34:46
Lol, Ok, take it down.


Don't skip looking at the link I posted in Reply #26; you'll like it.
Title: Re: False Vacuum; Who, What, Where, When, Why?
Post by: Bill S on 27/07/2018 12:55:23
Quote
Don't skip looking at the link I posted in Reply #26; you'll like it.

Had a quick look, should there have been sound?
Trying to link it to the OP.
Title: Re: False Vacuum; Who, What, Where, When, Why?
Post by: Bogie_smiles on 27/07/2018 14:01:58
Quote
Don't skip looking at the link I posted in Reply #26; you'll like it.


Had a quick look, should there have been sound?
Trying to link it to the OP.
The player accommodates movies with sound but there was no sound as far as I could tell.


For me, it links to the OP because it looks a lot like what I would envision as the type of random energy density fluctuations that could lead to the formation of a single bubble nucleating in a patch of true vacuum in QFT.

Title: Re: False Vacuum; Who, What, Where, When, Why?
Post by: Bogie_smiles on 28/07/2018 21:13:08
Reply #31


Let’s consider, now, a landscape in which there is a valley at a higher energy level than the minimum. This is where the scalar field comes to rest. We have not established any reason why it might, or might not, discharge its energy, but since tunneling is going to be considered, it must be that the field remains as a quon.
You mentioned this before when we were talking about “nothingness” and I have avoided it because I can’t find anything on the quon, except in the works of Vilekin.


How is it related to the false or true vacuum?  Do you think Alexander Vilekin is on to something, or is off somewhere? How does tunneling come into the picture in his books?


Still hoping someone will pick up on my view of what we observe in our known universe and help me relate it to the first few paragraphs of the QFT link to the Wiki on False Vacuum:
https://en.wikipedia.org/wiki/False_vacuum (https://en.wikipedia.org/wiki/False_vacuum)


In our Hubble field of view, we see a vast observable patch of space filled with galactic structure, and there is a shift toward the red in the spectrum of light coming from those galaxies that is convincing evidence that the components of that structure are moving away from us and each other, and the rate of that separation appears to be accelerating. Also, we observe a cosmic microwave background (CMB), and within that background there are two anomalies, the wide angle temperature difference, called hemispherical asymmetry or dipole anisotropy (http://iopscience.iop.org/article/10.1086/518091/meta (http://iopscience.iop.org/article/10.1086/518091/meta)), andthere is a large cold spot (https://en.wikipedia.org/wiki/CMB_cold_spot (https://en.wikipedia.org/wiki/CMB_cold_spot)) in the temperature readings from the WMAP and Planck surveys. Bill S didn’t take exception to that, but wanted me to relate it to the of the False vacuum.


The connection is that our observable universe has gone through a phenomenal decline in energy density since the initial expansion/inflation, and that links it to the hypotheticals about false and true vacuums. Inflation leads to cooling of the background and to the longer microwaves of the observed background, and the resulting lower density should support the period of more stable particles, which we are experiencing.


If the false vacuum is somewhat stable, and can evolve over a very long period of time to become more stable, that sounds like what our observable universe seems to have done. We could have moved from a state that equates to a local minimum (higher density), to a more stable state of lower density. Any agreement among the viewers, or helpful tips?
 
https://www.thenakedscientists.com/forum/gallery/43933_25_07_18_6_47_50.jpeg (https://www.thenakedscientists.com/forum/gallery/43933_25_07_18_6_47_50.jpeg)
(https://www.thenakedscientists.com/forum/gallery/43933_25_07_18_6_47_50.jpeg)
Title: Re: False Vacuum; Who, What, Where, When, Why?
Post by: Bill S on 29/07/2018 01:04:30
Sorry. Notime to respond to this tonight, but, as I said earlier, "quon" is not a word Vilenkin used. 

Here is the Wiki link.

https://en.wikipedia.org/wiki/Quon

I knew it was in Wiki because I put it there a few years ago.:)
Title: Re: False Vacuum; Who, What, Where, When, Why?
Post by: Bogie_smiles on 29/07/2018 02:10:48
Reply #33

Sorry. No time to respond to this tonight, but, as I said earlier, "quon" is not a word Vilenkin used. 

Here is the Wiki link.

https://en.wikipedia.org/wiki/Quon (https://en.wikipedia.org/wiki/Quon)

I knew it was in Wiki because I put it there a few years ago. :)
Sorry, I misunderstood.

I am on the trail of some perspective from searching the Internet to help me understand the issues involved in this topic, and of course it is important to share what we find, (and for you to share what you may already know) about the topic.

In regard to the content below, my conclusion is added for discussion at the end.

Quote
http://math.ucr.edu/home/baez/vacuum.html (http://math.ucr.edu/home/baez/vacuum.html)
Here’s the deal. We have two fundamental theories of physics: quantum field theory and general relativity.

Quantum field theory takes quantum mechanics and special relativity into account, and it's a great theory of all the forces and particles except gravity, but it ignores gravity.

General relativity is a great theory of gravity, but it ignores quantum mechanics.

Nobody knows how to reconcile these theories yet. That's what people working on "quantum gravity" are trying to do.

Now, the reason I'm telling you this is that quantum field theory and general relativity have really different attitudes towards the energy density of the vacuum. The reason is that quantum field theory only cares about energy differences. If you can only measure energy differences, you can't determine the energy density of the vacuum - it's just a matter of convention.

As far as we know, you can only determine the energy density of the vacuum by experiments that involve general relativity - namely, by measuring the curvature of spacetime.

So, when you ask about the energy density of the vacuum, you get different answers depending on whether the person answering you is basing their answer on general relativity or quantum field theory. Let me run through the 5 most common answers, explaining how people reach these different answers:

 
  • We can measure the energy density of the vacuum through astronomical observations that determine the curvature of spacetime. All the measurements that have been done agree that the energy density is VERY CLOSE TO ZERO. In terms of mass density, its absolute value is less than 10-26 kilograms per cubic meter. In terms of energy density, this is about 10-9 joules per cubic meter. One can know something is very close to zero without knowing whether it is positive, negative or zero. For a long time that's how it was with the cosmological constant. But, recent measurements by the Wilkinson Microwave Anisotropy Probe and many other experiments seem to be converging on a positive cosmological constant, equal to roughly 7 × 10-27 kilograms per cubic meter. This corresponds to a positive energy density of about 6 × 10-10 joules per cubic meter.
     The reason they get a positive energy density is very interesting. Thanks to the redshifts of distant galaxies and quasars, we've known for a long time that the universe is expanding. The new data shows something surprising: this expansion is speeding up. Ordinary matter can only make the expansion slow down, since gravity attracts - at least for ordinary matter.
     What can possibly make the expansion speed up, then? Well, general relativity says that if the vacuum has energy density, it must also have pressure! In fact, it must have a pressure equal to exactly -1 times its energy density, in units where the speed of light and Newton's gravitational constant equal 1. Positive energy density makes the expansion of the universe tend to slow down... but negative pressure makes the expansion tend to speed up.
     More precisely, the rate at which the expansion of the universe accelerates is proportional to
     - ρ - 3P
     where ρ is the energy density and P is the pressure. (This isn't supposed to be obvious: there's a nontrivial calculation involved, and I'm just telling you the final result. The 3 is there because there are 3 dimensions of space, oddly enough.)
     But as I mentioned, for the vacuum the pressure is minus the energy density: P = -ρ. So, the rate at which the vacuum makes the expansion of the universe accelerate is proportional to
     2 ρ
     From this, it follows that if the vacuum has positive energy density, the expansion of the universe will tend to speed up! This is what people see. And, vacuum energy is currently the most plausible explanation known for what's going on.
     Of course, to believe this argument at all, one must have some confidence in general relativity. To believe scientists' attempts to determine an actual value for the energy density of spacetime, one must have more confidence in general relativity, and also other assumptions about cosmology. However, the basic fact that the energy density of spacetime is very close to zero is almost unarguable: for it to be false, general relativity would have to be very wrong.
     
    We can try to calculate the energy density of the vacuum using quantum field theory. If we calculate the lowest possible energy of a harmonic oscillator, we get a bigger answer when we use quantum mechanics than when we use classical mechanics. The difference is called the "zero-point energy". The zero-point energy of a harmonic oscillator is 1/2 Planck's constant times its frequency. Naively we can try calculating the energy density of the vacuum by simply summing up the zero-point energies of all the vibrational modes of the quantum fields we are considering (e.g. the electromagnetic field and various other fields for other forces and particles). Vibrational modes with shorter wavelengths have higher frequencies and contribute more vacuum energy density. If we assume spacetime is a continuum, we have modes with arbitrarily short wavelengths, so we get INFINITY as the vacuum energy density. But there are problems with this calculation.... A slightly less naive way to calculate the vacuum energy in quantum field theory is to admit that we don't know spacetime is a continuum, and only sum the zero-point energies for vibrational modes having wavelengths bigger than, say, the Planck length (about 10-35 meters). This gives an ENORMOUS BUT FINITE vacuum energy density. Using E = mc2 to convert between energy and mass, it corresponds to a mass density of about 1096 kilograms per cubic meter! But there are problems with this calculation, too.... One problem is that treating the vibrational modes of our fields as harmonic oscillators is only valid for "free field theories" - those in which there are no interactions between modes. This is not physically realistic.
     However, while taking interactions into account changes the precise answer, we are still left with an enormous energy density. The ridiculous ratio between this density and what's actually observed is often called the cosmological constant problem. One way to put it is that in units of Planck mass per Planck length cubed, the cosmological constant is about 10
    -123. It's hard to make up a theory that explains such a tiny nonzero number.
     But there's an even bigger problem, too....
     
    Quantum field theory as it is ordinarily done ignores gravity. But as long as one is ignoring gravity, one can add any constant to ones definition of energy density without changing the predictions for anything you can experimentally measure. The reason is that without measuring the curvature of spacetime, one can only measure energy differences. The big problem with calculations 2 and 3 is that they ignore this fact. If we take advantage of this fact we are free to redefine energy density by subtracting off the zero-point energy, leaving an energy density of ZERO. In fact this is what is ordinarily done in quantum field theory. An even less naive way to think about the vacuum energy density in quantum field theory is the following. In quantum field theory we are neglecting gravity. This means we are free to add any constant whatsoever to our definition of energy density. As long as we are free to do this, we can't really say what the vacuum energy density "really is". In other words, if we only consider quantum field theory and not general relativity, the vacuum energy density is NOT DETERMINED.
So, I've given you 5 answers to the same question:
 
  • VERY CLOSE TO ZERO INFINITY ENORMOUS BUT FINITE ZERO NOT DETERMINED

Which should you believe? I believe
1) because it is based on experiment and fairly conservative assumptions about general relativity and astronomy.

Answers 2)-4) are based on somewhat naive theoretical calculations.

Answer 5) is the best that quantum field theory can do right now.

Reconciling answers 1) and 5) is one of the big tasks of any good theory of quantum gravity.

The moral is: for a question like this, you need to know not just the answer but also the assumptions and reasoning that went into the answer. Otherwise you can't make sense of why different people give different answers.


I’m reading Baez for the umpteenth time (and Ned Wright, too) over recent years, and I find that I understand him much better as I learn more GR and QFT; surprise surprise, right Gomer Pyle?

My effort to get the scenario of our observable universe and mainstream theory on the table as we explore the False Vacuum, is that, whether we invoke QFT or GR, we still are dealing with vacuum energy density; energy density in its role as a major factor in the EFEs on one hand, and in its role as a major factor in QFT on the other.

Let the chips fall where they will, but it just seems logical to insist on using the known observable universe as a major part of the decision process as we sort out which side to take on the issue.
Title: Re: False Vacuum; Who, What, Where, When, Why?
Post by: Bill S on 29/07/2018 16:57:56
Bogie_Smiles, I think you need someone with more expertise (and time) than I have to do justice to that lot.  :)
Title: Re: False Vacuum; Who, What, Where, When, Why?
Post by: Bogie_smiles on 29/07/2018 18:09:58
Bogie_Smiles, I think you need someone with more expertise (and time) than I have to do justice to that lot.  :)
First of all, I know what you mean by "that lot"; it is a lot of text to say basically that you either invoke General Relativity, or Quantum Field Theory, buy not both, because they don't play well together (yet).

I know what you mean by needing more time, because 1) if your are like me, if the material is not right in line with your interests, it is near impossible to find the time to wade through it, and 2) even if it is an interesting proposition, it does take a lot of time to go in that direction if you haven't already started down the path.

And I don't believe that you don't have the expertise to wade through it if you were inclined, but I don't expect anyone to have the inclination to pick it up and run with it if they aren't already into thinking about TOE and a quantum solution to gravity.

Thanks to you and those who have participated, and let's see if anyone else cares to jump in with any pros or cons about the false vacuum and QFT.
Title: Re: False Vacuum; Who, What, Where, When, Why?
Post by: Bill S on 29/07/2018 23:20:43
Quote from: Bogie_Smiles
I know what you mean by needing more time, because 1) if your are like me, if the material is not right in line with your interests, it is near impossible to find the time to wade through it, and 2) even if it is an interesting proposition, it does take a lot of time to go in that direction if you haven't already started down the path.

It's certainly not lack of interest on my part, but I care for two disabled people, and have a serious ILD which slows me down.  Some days it's a job to find time to open the computer.  This thread has made me think seriously about the vacuum.  Thanks for that.

We've both asked quite a lot of questions; let's hope they are answered.
Title: Re: False Vacuum; Who, What, Where, When, Why?
Post by: Bogie_smiles on 30/07/2018 01:23:54
It's certainly not lack of interest on my part, but I care for two disabled people, and have a serious ILD which slows me down.  Some days it's a job to find time to open the computer.  This thread has made me think seriously about the vacuum.  Thanks for that.

That can't be easy. Hopefully things will take a turn for the better. Hang in there and keep active on the forum when time and responsibilities permit.
Title: Re: False Vacuum; Who, What, Where, When, Why?
Post by: Bill S on 30/07/2018 12:16:20
Quote
Hang in there and keep active on the forum when time and responsibilities permit.

Will do.  I'd say it helps to ward of dementia, if I could remember what that was. :)

Thanks for the kind thoughts.
Title: Re: False Vacuum; Who, What, Where, When, Why?
Post by: Bill S on 30/07/2018 20:50:07
Thinking about the OP, I asked myself why cosmology needed an energetic vacuum in the first place.  My thinking went along these lines:   

 Most, if not all, models of an inflationary universe require the existence of a vacuum, with a measurable (non-zero) vacuum energy in order to have a situation in which this inflation can operate.  Quantum mechanics provides us with this type of vacuum, because it forbids us from having a classical vacuum, which can be identified as absolutely nothing.   Absolute nothing provides absolute information about its state, and the uncertainty principle does not give us that luxury. 

If the vacuum is a reality, and there is good reason to think it is, there has to be the possibility that the vacuum is “something”.  In fact, to the best of our knowledge, the vacuum is, on the scale of the Planck’s length, a very active and energetic place.   

An additional requirement (one that for reasons which I find inexplicable, some experts hesitate to agree with) is that either this vacuum must be eternal, or it must have emerged from something else. 
Title: Re: False Vacuum; Who, What, Where, When, Why?
Post by: jeffreyH on 30/07/2018 21:50:16
If you look at the staggered troughs of the true/false vacuum and compare this with the Higgs potential, then the Higgs mechanism would have to favour a false vacuum in order to supply a mass term. In which case the true vacuum may only contain particles with zero rest mass. If we actually exist within a true vacuum then no such bias is required. Occam's razor suggests the latter.
Title: Re: False Vacuum; Who, What, Where, When, Why?
Post by: Bill S on 30/07/2018 23:16:45
Quote from: Jeffrey
If you look at the staggered troughs of the true/false vacuum and compare this with the Higgs potential, then the Higgs mechanism would have to favour a false vacuum in order to supply a mass term.

Simple explanation, please, Jeffrey.
Title: Re: False Vacuum; Who, What, Where, When, Why?
Post by: Bogie_smiles on 31/07/2018 05:21:38



Reply #42






Quantum mechanics provides us with this type of vacuum, because it forbids us from having a classical vacuum, which can be identified as absolutely nothing.
We are on the same wavelength, and share disdain for “nothingness” :)

But I might like to nit pick the part about why QFT provides us with energy density everywhere. Not that I don’t think it is true that QFT denies us nothingness, but perhaps the reason is that a total or perfect vacuum cannot be achieved simply because there is no empty space; field is everywhere. 

If so … the vacuum has energy density everywhere, and the false vacuum has higher energy density than the true vacuum, but the true vacuum cannot be void of energy.
Quote




If the vacuum is a reality, and there is good reason to think it is, there has to be the possibility that the vacuum is “something”.  In fact, to the best of our knowledge, the vacuum is, on the scale of the Planck’s length, a very active and energetic place.

An additional requirement (one that for reasons which I find inexplicable, some experts hesitate to agree with) is that either this vacuum must be eternal, or it must have emerged from something else.


So we are in agreement again. The field is quite active at the quantum level, and in fact one conclusion offered by the Baez page in reply #33 was clear that one very active issue in the scientific community is to find a quantum solution to gravity. For example he said, “Here’s the deal. We have two fundamental theories of physics: quantum field theory and general relativity. … Quantum field theory takes quantum mechanics and special relativity into account, and it's a great theory of all the forces and particles except gravity, but it ignores gravity. … General relativity is a great theory of gravity, but it ignores quantum mechanics. … Nobody knows how to reconcile these theories yet. That's what people working on "quantum gravity" are trying to do.”

Title: Re: False Vacuum; Who, What, Where, When, Why?
Post by: jeffreyH on 31/07/2018 21:19:08
The Higgs field would have to be tuned precisely to the value of the false vacuum for it not to have a detectable effect upon it. See the following.
https://profmattstrassler.com/articles-and-posts/particle-physics-basics/how-the-higgs-field-works-with-math/2-why-the-higgs-field-is-non-zero-on-average/
Title: Re: False Vacuum; Who, What, Where, When, Why?
Post by: jeffreyH on 01/08/2018 18:59:17
You could explain virtual particles in terms of quantum oscillations between the true and false vacuum states. This cannot be thought of in terms of tunneling, since this does not involve real particles. This speculation likely falls down for various reasons. I don't know enough to judge. Maybe others can comment.
Title: Re: False Vacuum; Who, What, Where, When, Why?
Post by: Bogie_smiles on 02/08/2018 09:37:02
Reply #45


You could explain virtual particles in terms of quantum oscillations between the true and false vacuum states. This cannot be thought of in terms of tunneling, since this does not involve real particles. This speculation likely falls down for various reasons. I don't know enough to judge. Maybe others can comment.
Discussing virtual particles is part of the legitimate discussion of particle interactions and the nature of transition from the false vacuum at a particular energy and a higher of lower energy level of the vacuum. I will post related links to Wikis below, but whether the casual reader wants to indulge in reading that level of detail or not, they are intended to support these comments in this discussion of virtual particles in regard to the topic of the False Vacuum.

Stated earlier: “Therefore, the transition to the true vacuum must be stimulated by the creation of high-energy particles (https://en.wikipedia.org/wiki/Particle_physics) or through quantum-mechanical tunneling (https://en.wikipedia.org/wiki/Quantum_tunnelling)."

Your post brings to mind just what the vacuum is composed of; we have a lot of energy in a scalar field of  QFT, which includes the energy of a gravitational field. Virtual particles conserve energy and momentum while they are present, and virtual photons come into play in interactions involving particle scattering and Casimir forces (https://en.wikipedia.org/wiki/Casimir_force). In quantum field theory, even classical forces—such as the electromagnetic repulsion (https://en.wikipedia.org/wiki/Electromagnetic_repulsion) or attraction between two charges—can be thought of as due to the exchange of many virtual photons between the charges.

This thread legitimately can describe the energy of the false vacuum as including virtual particles, with the stipulation that they aren’t the same as the fundamental particles of the standard model, and don’t necessarily have the same mass as a fundamental particle, but they can exist in the scalar field of the false vacuum, and as such, can be part of the energy involved in moving from a false vacuum to a true vacuum.

Comment freely and certainly state any objections to that stated position anyone might have.

__________________________________________________
Related links and quotes from:

https://en.wikipedia.org/wiki/Particle_physics (https://en.wikipedia.org/wiki/Particle_physics)
Particle physics (also high energy physics) is the branch of physics that studies the nature of the particles that constitute matter and radiation. Although the word particle can refer to various types of very small objects (e.g. protons, gas particles, or even household dust), particle physics usually investigates the irreducibly smallest detectable particles and the fundamental interactions necessary to explain their behaviour. By our current understanding, these elementary particles are excitations of the quantum fields that also govern their interactions. The currently dominant theory explaining these fundamental particles and fields, along with their dynamics, is called the Standard Model. Thus, modern particle physics generally investigates the Standard Model and its various possible extensions, e.g. to the newest "known" particle, the Higgs boson, or even to the oldest known force field, gravity.

“particle physics usually investigates the irreducibly smallest detectable particles and the fundamental interactions necessary to explain their behaviour.”

Then a look at:
https://en.wikipedia.org/wiki/Virtual_particle (https://en.wikipedia.org/wiki/Virtual_particle)
In physics, a virtual particle is a transient fluctuation that exhibits some of the characteristics of an ordinary particle, but whose existence is limited by the uncertainty principle. The concept of virtual particles arises in perturbation theory of quantum field theory where interactions between ordinary particles are described in terms of exchanges of virtual particles. Any process involving virtual particles admits a schematic representation known as a Feynman diagram, in which virtual particles are represented by internal lines.[1][2]Virtual particles do not necessarily carry the same mass as the corresponding real particle, although they always conserve energy and momentum. The longer the virtual particle exists, the closer its characteristics come to those of ordinary particles. They are important in the physics of many processes, including particle scattering and Casimir forces. In quantum field theory, even classical forces—such as the electromagnetic repulsion or attraction between two charges—can be thought of as due to the exchange of many virtual photons between the charges.
The term is somewhat loose and vaguely defined, in that it refers to the view that the world is made up of "real particles": it is not; rather, "real particles" are better understood to be excitations of the underlying quantum fields. Virtual particles are also excitations of the underlying fields, but are "temporary" in the sense that they appear in calculations of interactions, but never as asymptotic states or indices to the scattering matrix. The accuracy and use of virtual particles in calculations is firmly established, but as they cannot be detected in experiments, deciding how to precisely describe them is a topic of debate.
Properties[edit]
The concept of virtual particles arises in the perturbation theory of quantum field theory, an approximation scheme in which interactions (in essence, forces) between actual particles are calculated in terms of exchanges of virtual particles. Such calculations are often performed using schematic representations known as Feynman diagrams, in which virtual particles appear as internal lines. By expressing the interaction in terms of the exchange of a virtual particle with four-momentum q, where q is given by the difference between the four-momenta of the particles entering and leaving the interaction vertex, both momentum and energy are conserved at the interaction vertices of the Feynman diagram.[3]:119
A virtual particle does not precisely obey the energy–momentum relationm2c4 = E2p2c2. Its kinetic energy may not have the usual relationship to velocity–indeed, it can be negative.[4]:110 This is expressed by the phrase off mass shell (https://en.wikipedia.org/wiki/On_shell_and_off_shell).[3]:119 The probability amplitude for a virtual particle to exist tends to be canceled out by destructive interference (https://en.wikipedia.org/wiki/Destructive_interference) over longer distances and times. As a consequence, a real photon is massless and thus has only two polarization states, whereas a virtual one, being effectively massive, has three polarization states.
Quantum tunnelling (https://en.wikipedia.org/wiki/Quantum_tunnelling) may be considered a manifestation of virtual particle exchanges.[5]:235 The range of forces carried by virtual particles is limited by the uncertainty principle, which regards energy and time as conjugate variables; thus, virtual particles of larger mass have more limited range.[6]

Quantum tunnelling may be considered a manifestation of virtual particle exchanges.[5]:235 The range of forces carried by virtual particles is limited by the uncertainty principle, which regards energy and time as conjugate variables; thus, virtual particles of larger mass have more limited range.”
Title: Re: False Vacuum; Who, What, Where, When, Why?
Post by: Bill S on 04/08/2018 19:16:50
Thoughts about an energetic vacuum.  Absence makes the thoughts grow weirder!

I keep finding my thoughts back at the OP, and wondering if I can find time to pick through this thread to see if the answers to my inevitable questions are in there somewhere.  If they are, please forgive me, but I’m going to ask for some responses to these thoughts.

It makes no sense to talk of an absolute vacuum filling space.  An absolute vacuum is nothing, so it can fill nothing. 

As soon as one starts to think of a vacuum as having qualities, the concept changes, the vacuum becomes something, and has to be somewhere.  If the “vacuum of space” has energy, it is something; so, where is it? 

 Instinctively, the answer would seem to be “everywhere”; it fills the Universe.

Would it be more meaningful to say that it is the Universe?  This would mean that what we perceive as matter and energy are physical manifestations of the all-pervading vacuum energy.

Further consideration of the idea that the vacuum energy fills the Universe must lead one to ask if, in fact, it goes beyond that. 

If we accept that the Universe came into being at the Big Bang; and if we accept inflationary theory, which I am doing for the purposes of this line of thought, then our Universe began as a “quon-sized” scalar field, “rolling about” on a hypothetical landscape that represents the vacuum energy. 

It would follow from this that the energetic vacuum must have preceded the existence of the Universe.

Can we say anything about the possible extent of the vacuum, in this case?

Must it be infinite?

Must it always have existed?

Is there any alternative to either of those?
Title: Re: False Vacuum; Who, What, Where, When, Why?
Post by: Bill S on 04/08/2018 22:43:28
Trying to make a start on #45.

Quote from: Bogie_Smiles
Virtual particles conserve energy and momentum while they are present

So Wiki assures us, but here we are talking cosmic scale.  Is energy necessarily conserved in an expanding universe?
Title: Re: False Vacuum; Who, What, Where, When, Why?
Post by: Bogie_smiles on 04/08/2018 23:50:27
Reply #48


Thoughts about an energetic vacuum.  Absence makes the thoughts grow weirder!

I keep finding my thoughts back at the OP, and wondering if I can find time to pick through this thread to see if the answers to my inevitable questions are in there somewhere.  If they are, please forgive me, but I’m going to ask for some responses to these thoughts.

It makes no sense to talk of an absolute vacuum filling space.  An absolute vacuum is nothing, so it can fill nothing. 

As soon as one starts to think of a vacuum as having qualities, the concept changes, the vacuum becomes something, and has to be somewhere.  If the “vacuum of space” has energy, it is something; so, where is it? 

Instinctively, the answer would seem to be “everywhere”; it fills the Universe.

Would it be more meaningful to say that it is the Universe?  This would mean that what we perceive as matter and energy are physical manifestations of the all-pervading vacuum energy.

Further consideration of the idea that the vacuum energy fills the Universe must lead one to ask if, in fact, it goes beyond that. 

If we accept that the Universe came into being at the Big Bang; and if we accept inflationary theory, which I am doing for the purposes of this line of thought, then our Universe began as a “quon-sized” scalar field, “rolling about” on a hypothetical landscape that represents the vacuum energy. 

It would follow from this that the energetic vacuum must have preceded the existence of the Universe.

Can we say anything about the possible extent of the vacuum, in this case?

Must it be infinite?

Must it always have existed?

Is there any alternative to either of those?

I don’t think those thoughts are weird at all. Thanks for weighing in with them, and I too have similar questions.

I’m at the point where I consider ruling out the possibility of an absolute vacuum too. Nothingness is the alternative, and you know how I feel about “something from nothing”.

To me, given the big bang event, and in line with the language of the topic, I’m wondering why it wouldn’t seem appropriate to think that our big bang was preceded by a bubble nucleation event within a false vacuum?
Quote from: Wiki
In the theoretical physics of the false vacuum, the system moves to a lower energy state – either the true vacuum, or another, lower energy vacuum – through a process known as bubble nucleation (https://en.wikipedia.org/wiki/Nucleation).

I don’t see any language about a beginning of the true vacuum. It doesn’t seem to be implied that bubble nucleation arises from nothingness; it arises from the false vacuum as a new higher energy bubble nucleates within a false vacuum, so if bubble nucleation can equate to the kind of event that we have come to think of as a big bang, the preconditions to a big bang event would be a relatively low energy false vacuum.

I also hate the talk that equates bubble nucleation, big bangs in the context of this individual post, with new universes. The bubbles nucleate in QFT to become expanding bubbles within a landscape of false and true vacuums, full of nucleated bubbles that start with low entropy and expand as entropy progresses (equating expansion to inflation). Why can't it be thought of as all one universe full of various vacuum densities and nucleated bubbles?

As for matter, I’m reminded of the statement that an event where bubbles converge or collide, matter occurs at the site:
Quote from: Wiki
If two bubbles are nucleated and they eventually collide, it is thought that particle production would occur where the walls collide.

There are many correlations between BBT and QFT, but the force of gravity seems to be the deciding factor. BBT gets gravity from the curvature of space, and QFT needs a quantum solution to gravity in order to come of age, so to speak.
Title: Re: False Vacuum; Who, What, Where, When, Why?
Post by: Bogie_smiles on 05/08/2018 01:37:11

Trying to make a start on #45.

Quote from: Bogie_Smiles
Virtual particles conserve energy and momentum while they are present


So Wiki assures us, but here we are talking cosmic scale.  Is energy necessarily conserved in an expanding universe?


In an expanding universe, is energy necessarily conserved?

I know of no particle interactions or energy events where energy is not conserved. However, if the origin of matter and energy was the big bang event, and if we equate that event to the start of the universe out of nothing, for the sake of discussion, then there is no reason to expect that energy that can theoretically come from nothingness would necessarily be conserved. 

In a single nucleated bubble in a QFT landscape, where the origin of the energy is part of the history of the false and real vacuums of the landscape, a bubble that is expanding in the false vacuum may be a different story, i.e., what indication is there that energy is not conserved?
Title: Re: False Vacuum; Who, What, Where, When, Why?
Post by: Bill S on 05/08/2018 14:03:22
Quote from: Bogie_Smiles
I know of no particle interactions or energy events where energy is not conserved.


Nor do I; but if you are linking virtual particle interactions to expansion of the Universe, then it might be appropriate to remember that energy is not necessarily conserved.
Title: Re: False Vacuum; Who, What, Where, When, Why?
Post by: Bogie_smiles on 05/08/2018 19:04:42
Reply #51


Quote from: Bogie_Smiles
I know of no particle interactions or energy events where energy is not conserved.

Nor do I; but if you are linking virtual particle interactions to expansion of the Universe (italics added), then it might be appropriate to remember that energy is not necessarily conserved.

I agree, linking QFT virtual particle interactions to expansion would be a risky and presumptuous game, and in my comment to jeffreyH I was just lending support to his post, based on repeating from the links about the false vacuum that I provided. The comment should be confined to the presence of virtual particles, when they pop into and out of existence, presumable from the energy of the local vacuum.

There is a tendency to equate the use of the phrase “expansion of the Universe” to the implication that the whole universe is expanding, when actually only a portion of the space that is referred to as “causally connected to our big bang event” is visible in our Hubble view.

That same tendency to refer to expansion of the Universe seems less evident in regard to QFT, perhaps because QFT boldly supports multiple bubbles nucleating in the false vacuums, while BBT with inflation seems more comfortable referring to a single event. Is it just me, or is there an inclination to equate such bubbles in QFT to some version of the initial events related to Big Bang Theory with Inflation?

The observable portion of the universe does show that for as far as we can see, distant galaxies are moving away from us and from each other, based the redshift data. That observed separation of galactic structure makes perfect sense if it is true that the stable atoms that formed within our causally connected big bang space occurred at around 300,000 years after the initial event (reference the formation of hydrogen atoms and the “surface of last scattering”). If all of the particles that make up the now distant and receding galactic structure were formed in the very early stages after the big bang, then it is reasonable that they would have separation momentum that is conserved as they accumulate to construct the now visible galactic structure (reference dark energy and the conservation of momentum). As far as I know, that idea is in accord with observations of known science, and is an appropriate argument supporting the consensus view of the cause of the redshift data as it pertains to GR and QFT.
Title: Re: False Vacuum; Who, What, Where, When, Why?
Post by: jeffreyH on 05/08/2018 22:59:16
The thing missing in the conversation is zero point energy. How does this relate to the idea of a false vacuum? This is considered to be a ground state. Is this a false ground state?
Title: Re: False Vacuum; Who, What, Where, When, Why?
Post by: Bogie_smiles on 06/08/2018 00:35:38
Reply #53

The thing missing in the conversation is zero point energy. How does this relate to the idea of a false vacuum? This is considered to be a ground state. Is this a false ground state?
That is a great introduction to the ZPE topic, and I have a couple of links that can serve as a beginning for current research on the topic:

https://rationalwiki.org/wiki/Zero-point_energy (https://rationalwiki.org/wiki/Zero-point_energy)
https://en.wikipedia.org/wiki/Zero-point_energy (https://en.wikipedia.org/wiki/Zero-point_energy)

From the rationalwiki:
Quantum mechanics (https://rationalwiki.org/wiki/Quantum_mechanics), via Heisenberg's uncertainty principle, requires that the absolute position and velocity of any particle cannot both be simultaneously definable. From this it is an inevitable conclusion that even at a temperature of absolute zero, any substance must have a certain minimum energy. This energy is referred to as zero-point energy. The vacuum energy is the zero-point energy of all the fields in space — the electromagnetic field, other gauge fields, fermionic fields and the Higgs field.
There have been speculations that usable energy might be extracted using this energy as a source using something called the Casimir effect, but this is almost certainly pseudoscience, as it would require using an extremely large collector device similar in some ways to an electronic capacitor, but much larger and thinner, with a vacuum dielectric; there is no knowledge currently extant that can allow the creation of such a collector cell, and even if it was possible, the zero point energy in a volume the size of the Earth is so small as to be fairly useless.[1]
According to Martin Gardner in his essay collection Did Adam and Eve Have Navels?, one of the chief researchers involved in ZPE study is Harold Puthoff, who was one of the most prominent of a crowd of Geller-gawkers in the 1970s height of the Human Potential Movement. Gardner noted that Puthoff's work on ZPE lacked transparency and solid science, and in that regard was much like his Geller work.
—————————

From the Wikipedia:

Zero-point energy (ZPE) is the difference between the lowest possible energy, that a quantum mechanical system may have, and the classical minimum energy of the system. Unlike in classical mechanics, quantum systems constantly fluctuate in their lowest energy state due to the Heisenberg uncertainty principle.[1] As well as atoms and molecules, the empty space of the vacuum has these properties. According to quantum field theory, the universe can be thought of not as isolated particles but continuous fluctuating fields: matter fields, whose quanta are fermions (i.e. leptons and quarks), and force fields, whose quanta are bosons (e.g. photons and gluons). All these fields have zero-point energy.[2] These fluctuating zero-point fields lead to a kind of reintroduction of an aether in physics,[1][3] since some systems can detect the existence of this energy. However this aether cannot be thought of as a physical medium if it is to be Lorentz invariant such that there is no contradiction with Einstein's theory of special relativity.[1]


Comment freely ...
Title: Re: False Vacuum; Who, What, Where, When, Why?
Post by: Bill S on 06/08/2018 18:30:37
Quote from: Bogie_Smiles
  …The comment should be confined to the presence of virtual particles, when they pop into and out of existence, presumable from the energy of the local vacuum….

This is my simplistic understanding:

1. Energy is borrowed from the vacuum energy to bring a perturbation into existence.  This is identified as a virtual particle.

2. Repayment of the energy is to the vacuum.  Some authors say it must be replaced “before the vacuum misses it”.  I would question this.

3. The vacuum is an integral feature of the Universe.

4. If it were possible to observe the energy while it was being borrowed, it might not be where classical physics says it should be, if classical physics could be applied, here.  However, it must still be in the Universe.

5. At no time does energy leave or enter the universe; therefore, conservation of energy is not violated.  Any violation of energy conservation in an expanding universe does not arise from virtual particle creation/annihilation.  (Sticking my neck out!)

Quote
There is a tendency to equate the use of the phrase “expansion of the Universe” to the implication that the whole universe is expanding, when actually only a portion of the space that is referred to as “causally connected to our big bang event” is visible in our Hubble view.

As far as I am aware, we have no direct evidence of anything that is not “causally connected to our big bang event”.  It would seem reasonable to suggest that anything “beyond” is either expanding with the observed Universe; or is not in direct/active contact with it.

Quote
Is it just me, or is there an inclination to equate such bubbles in QFT to some version of the initial events related to Big Bang Theory with Inflation?

It’s not just you!
Title: Re: False Vacuum; Who, What, Where, When, Why?
Post by: Bill S on 06/08/2018 18:32:49
Here's one I thought earlier.

Ref. #10: 

 If/when a scalar field settles in a “valley”, what determines whether it remains or discharges its energy?
Title: Re: False Vacuum; Who, What, Where, When, Why?
Post by: Bogie_smiles on 07/08/2018 17:11:01
Reply #56

Let me address your last two posts together:
Quote from: Bogie_Smiles
  …The comment should be confined to the presence of virtual particles, when they pop into and out of existence, presumable from the energy of the local vacuum….

This is my simplistic understanding:

1. Energy is borrowed from the vacuum energy to bring a perturbation into existence.  This is identified as a virtual particle.

2. Repayment of the energy is to the vacuum.  Some authors say it must be replaced “before the vacuum misses it”.  I would question this.

3. The vacuum is an integral feature of the Universe.

4. If it were possible to observe the energy while it was being borrowed, it might not be where classical physics says it should be, if classical physics could be applied, here.  However, it must still be in the Universe.

5. At no time does energy leave or enter the universe; therefore, conservation of energy is not violated.  Any violation of energy conservation in an expanding universe does not arise from virtual particle creation/annihilation.  (Sticking my neck out!)

Quote
There is a tendency to equate the use of the phrase “expansion of the Universe” to the implication that the whole universe is expanding, when actually only a portion of the space that is referred to as “causally connected to our big bang event” is visible in our Hubble view.

As far as I am aware, we have no direct evidence of anything that is not “causally connected to our big bang event”.  It would seem reasonable to suggest that anything “beyond” is either expanding with the observed Universe; or is not in direct/active contact with it.

Quote
Is it just me, or is there an inclination to equate such bubbles in QFT to some version of the initial events related to Big Bang Theory with Inflation?

It’s not just you!


Here's one I thought earlier.

Ref. #10: 

If/when a scalar field settles in a “valley”, what determines whether it remains or discharges its energy?
Those replies show very astute analysis, IMHO.

The links we have looked at, and that are connected to them, tell us much about the theory of QFT. My take so far from the material and from generally accepted scientific observations and data about our known universe (influenced by the conclusion that we may well be in an expanding QFT bubble within the greater universe) is that the greater vacuum is an active place of bubble nucleation, bubble collisions, particle formation at the locations in the vacuum where those collisions occur, and expansion of resulting nucleated bubbles, governed by their vacuum density and by the vacuum density of the surrounding false/real vacuums.

The determining factors that govern the disposition and future course of events related to the energy contained in the vacuum of a particular bubble that settles in a “valley” is the relative vacuum energy density surrounding that bubble. That determination should have causes and limiting factors. A causative factor would be random nucleation and subsequent collisions of bubbles. Limiting factors would have to do with how the local vacuum densities react to the presence of adjacent bubbles and their vacuum densities.

The results of those possible events, including the collisions, the particle formation and nucleosynthesis at the boundaries of those collisions, and the evolution of galactic structure moving apart as the space occupied by the expanding bubble grows, all seem to nicely equate with what we observe in our Hubble view.

(https://www.thenakedscientists.com/forum/gallery/43933_08_08_18_1_00_04.jpeg)
Bubble Nucleation in QFT

Edit:
I have added a link to this content to my thread in New Theories called "If there was one Big Bang event, why not multiple big bangs?"
https://www.thenakedscientists.com/forum/index.php?topic=70348.msg550474#msg550474 (https://www.thenakedscientists.com/forum/index.php?topic=70348.msg550474#msg550474)

Title: Re: False Vacuum; Who, What, Where, When, Why?
Post by: Bogie_smiles on 09/08/2018 15:39:05
Reply #57

"In quantum field theory, a false vacuum is a hypothetical vacuum that is somewhat, but not entirely, stable." I don't believe anything to be entirely stable, so is it just a time frame thing? https://en.wikipedia.org/wiki/False_vacuum (https://en.wikipedia.org/wiki/False_vacuum)
I am continuing to refer to the Link provided by mrsmith2211 (https://en.wikipedia.org/wiki/False_vacuum (https://en.wikipedia.org/wiki/False_vacuum)).


Click on the link again, and right after the True vs. False Vacuum section comes the section on the Standard Model vacuum, which we really have to address in more detail now, if anyone is going to follow the possible scenario that I included in reply #56. In that post, I concluded that our current Hubble view seems to correspond to a mature stage of evolution of the collision between two nucleated bubbles. The collision (Big Bang?) would account for the formation of particles, and the expansion of the nucleated bubble that we occupy (Inflation) would account for the separation of those particles and of the structures that subsequently formed from them (galactic structure that we observe).

Therefore, separation and the conservation of momentum would account for the observation that galaxies and galaxy groups are generally moving away from us and from each other, and that would explain the observed redshift:
Redshift
In physics, redshift happens when light or other electromagnetic radiation from an object is increased in wavelength, or shifted to the red end of the spectrum.
(click for link to Redshift)
https://en.wikipedia.org/wiki/Redshift (https://en.wikipedia.org/wiki/Redshift)

Standard Model Vacuum from Wiki:
If the Standard Model (https://en.wikipedia.org/wiki/Standard_Model) is correct, the particles (https://en.wikipedia.org/wiki/Elementary_particle) and forces (https://en.wikipedia.org/wiki/Fundamental_force) we observe in our universe exist as they do because of underlying quantum fields (https://en.wikipedia.org/wiki/Quantum_field). Quantum fields can have states of differing stability, including 'stable', 'unstable', or 'metastable (https://en.wikipedia.org/wiki/Metastability)' (meaning very long-lived but not completely stable). If a more stable vacuum state were able to arise, then existing particles and forces would no longer arise as they do in the universe's present state. Different particles or forces would arise from (and be shaped by) whatever new quantum states arose. The world we know depends upon these particles and forces, so if this happened, everything around us, from subatomic particles (https://en.wikipedia.org/wiki/Subatomic_particle) to galaxies (https://en.wikipedia.org/wiki/Galaxy), and all fundamental forces (https://en.wikipedia.org/wiki/Fundamental_forces), would be reconstituted into new fundamental particles and forces and structures. The universe would lose all of its present structures and become inhabited by new ones (depending upon the exact states involved) based upon the same quantum fields.


Everyone is invited to participate with generally accepted science or detail about competing theories, but please review the Wiki piece above about the Standard Model Vacuum and take a look at the imbedded hyperlinks, which I want to include in the content of my thread.

Comment freely.

Edit: We have a lot of material to cover even if we stick to just the @mrsmith2211 link and the associated imbedded links.
Title: Re: False Vacuum; Who, What, Where, When, Why?
Post by: Bogie_smiles on 10/08/2018 00:18:40
Reply #58

The first link in the content about the Standard Model Vacuum posted in reply #57 goes to a more detailed Wiki page about the Standard Model. The imbedded links in that Wiki also have important content that we should review and share in order to stay with the direction of progress of the thread:


https://en.wikipedia.org/wiki/Standard_Model (https://en.wikipedia.org/wiki/Standard_Model):

(https://www.thenakedscientists.com/forum/gallery/43933_22_09_17_2_10_17.png)

The Standard Model of particle physics (https://en.wikipedia.org/wiki/Particle_physics) is the theory describing three of the four known fundamental forces (https://en.wikipedia.org/wiki/Fundamental_force) (the electromagnetic (https://en.wikipedia.org/wiki/Electromagnetism), weak (https://en.wikipedia.org/wiki/Weak_interaction), and strong (https://en.wikipedia.org/wiki/Strong_interaction) interactions, and not including the gravitational force (https://en.wikipedia.org/wiki/Gravity)) in the universe (https://en.wikipedia.org/wiki/Universe), as well as classifying all known elementary particles (https://en.wikipedia.org/wiki/Elementary_particle). It was developed in stages throughout the latter half of the 20th century, through the work of many scientists around the world,[1] (https://en.wikipedia.org/wiki/Standard_Model#cite_note-1) with the current formulation being finalized in the mid-1970s upon experimental confirmation (https://en.wikipedia.org/wiki/Experimental_confirmation) of the existence of quarks (https://en.wikipedia.org/wiki/Quark). Since then, confirmation of the top quark (https://en.wikipedia.org/wiki/Top_quark) (1995), the tau neutrino (https://en.wikipedia.org/wiki/Tau_neutrino) (2000), and the Higgs boson (https://en.wikipedia.org/wiki/Higgs_boson) (2012) have added further credence to the Standard Model. In addition, the Standard Model has predicted various properties of weak neutral currents (https://en.wikipedia.org/wiki/Weak_neutral_current) and the W and Z bosons (https://en.wikipedia.org/wiki/W_and_Z_bosons) with great accuracy.
Although the Standard Model is believed to be theoretically self-consistent[2] (https://en.wikipedia.org/wiki/Standard_Model#cite_note-2) and has demonstrated huge successes in providing experimental predictions (https://en.wikipedia.org/wiki/Experimental_prediction), it leaves some phenomena unexplained (https://en.wikipedia.org/wiki/Physics_beyond_the_standard_model) and falls short of being a complete theory of fundamental interactions (https://en.wikipedia.org/wiki/Theory_of_everything). It does not fully explain baryon asymmetry (https://en.wikipedia.org/wiki/Baryon_asymmetry), incorporate the full theory of gravitation (https://en.wikipedia.org/wiki/Theory_of_gravitation)[3] (https://en.wikipedia.org/wiki/Standard_Model#cite_note-DarkMatter-3) as described by general relativity (https://en.wikipedia.org/wiki/General_relativity), or account for the accelerating expansion of the Universe (https://en.wikipedia.org/wiki/Accelerating_expansion_of_the_Universe) as possibly described by dark energy (https://en.wikipedia.org/wiki/Dark_energy). The model does not contain any viable dark matter (https://en.wikipedia.org/wiki/Dark_matter) particle that possesses all of the required properties deduced from observational cosmology (https://en.wikipedia.org/wiki/Physical_cosmology). It also does not incorporate neutrino oscillations (https://en.wikipedia.org/wiki/Neutrino_oscillation) and their non-zero masses.
The development of the Standard Model was driven by theoretical (https://en.wikipedia.org/wiki/Theoretical_physics) and experimental (https://en.wikipedia.org/wiki/Experimental_physics) particle physicists alike. For theorists, the Standard Model is a paradigm of a quantum field theory (https://en.wikipedia.org/wiki/Quantum_field_theory), which exhibits a wide range of physics including spontaneous symmetry breaking (https://en.wikipedia.org/wiki/Spontaneous_symmetry_breaking), anomalies (https://en.wikipedia.org/wiki/Anomaly_(physics)) and non-perturbative behavior. It is used as a basis for building more exotic models that incorporate hypothetical particles (https://en.wikipedia.org/wiki/Hypothetical_particle), extra dimensions (https://en.wikipedia.org/wiki/Extra_dimensions), and elaborate symmetries (such as supersymmetry (https://en.wikipedia.org/wiki/Supersymmetry)) in an attempt to explain experimental results at variance with the Standard Model, such as the existence of dark matter and neutrino oscillations.
[End of quote from Wiki on Standard Model]

Take note of this particular link in the Standard Model Wiki quote labeled “Universe”.
https://en.wikipedia.org/wiki/Universe (https://en.wikipedia.org/wiki/Universe)
It is a topic that was discussed in my previous thread about “nothingness”, and its logical opposite, “universe” which is all of space and time and their contents.It would be nice to be on the same page in regard to the concept of “Universe”; and that would be the page that says there is only one universe. If you have a different view, I am interested in the “who, what, where, when and why” behind what you're thinking. Why do we need to have a concept that there are multiple universes? The QFT thinking that multiple bubbles nucleating within the true and false vacuums of space is sufficient, and allows for as many bubble nucleations as you want, and therefore as many big bangs as you want, if you equate collisions of nucleating bubbles with matter producing big bang type events as proposed in QFT.

Comment freely …
Title: Re: False Vacuum; Who, What, Where, When, Why?
Post by: Bogie_smiles on 10/08/2018 11:02:40
Reply #59

Standard Model Vacuum from Wiki:

If the Standard Model (https://en.wikipedia.org/wiki/Standard_Model) is correct, the particles (https://en.wikipedia.org/wiki/Elementary_particle) and forces (https://en.wikipedia.org/wiki/Fundamental_force) we observe in our universe exist as they do because of underlying quantum fields (https://en.wikipedia.org/wiki/Quantum_field). Quantum fields can have states of differing stability, including 'stable', 'unstable', or 'metastable (https://en.wikipedia.org/wiki/Metastability)' (meaning very long-lived but not completely stable). If a more stable vacuum state were able to arise, then existing particles and forces would no longer arise as they do in the universe's present state. Different particles or forces would arise from (and be shaped by) whatever new quantum states arose. The world we know depends upon these particles and forces, so if this happened, everything around us, from subatomic particles (https://en.wikipedia.org/wiki/Subatomic_particle) to galaxies (https://en.wikipedia.org/wiki/Galaxy), and all fundamental forces (https://en.wikipedia.org/wiki/Fundamental_forces), would be reconstituted into new fundamental particles and forces and structures. The universe would lose all of its present structures and become inhabited by new ones (depending upon the exact states involved) based upon the same quantum fields.
That makes perfect sense, and corresponds nicely with Big Bang theory with inflation.

If we backtrack the currently expanded state of our observable universe, going back in time we would get to an initial event. Under QFT that event could be caused by the colliding nucleating bubble “walls” or perhaps what we could call wave fronts, that result in the production of particles. The collisions, coupled with at least speed of light “bubble expansion” that one should expect to be associated with of the velocity of electromagnetism in the quantum field, the extreme high energy density vacuum at the outset would quickly evolve, resulting in the advent of the more stable vacuum states for particles. That means that as various sequences of massive particles (exotics relative to the standard model of particle physics as far as we know) form and decay in correspondence with the rapidly declining vacuum energy density of the local environment as the nucleated bubble evolves, longer periods of stability should be expected along with the normal course of particle decay.

That course of decay would continue until the resulting vacuum density can support the stability of the resulting particles for an extended period of time, as is the case with our own experience as an expanding bubble after some 14 billion years; one of a potentially infinite number of nucleated bubbles within the one greater universe :).

Comment freely.

Title: Re: False Vacuum; Who, What, Where, When, Why?
Post by: Bogie_smiles on 11/08/2018 00:24:59
Reply #60

In the last post I referred to expanding bubbles as wave fronts, and related the velocity of bubble expansion to an event in the QFT Wiki called bubble collisions. Bubble collisions are supposed to produce particles and matter, and following the QFT scenario, it would seem safe to equate that to our own circumstance. That would mean that bubble collisions account for the production of particles, and set particles into motion.

Photons have to be part of the particle mix, and so it can be concluded that it would be consistent with QFT that the bubble collisions would set photons into motion in all directions through the vacuum energy density of space. That could be equated to an expanding wave of electromagnetic energy traversing the quantum field right along with the bubble expansion.

That is not intended to equate the bubble wall to the advance of photon energy, but from what I have read, I would expect the expanding bubble to be filled with light, and thus with photon energy. However, the quantum field is not just an electromagnetic field, it is all fields, and all forms of energy expand through it according to their individual fields. The electric field and the magnetic field combine to govern the speed of light. …


There is not yet a quantum field solution to gravity, so when we talk theory of the universe, we have to rely on General Relativity for now, which is a macro level force. Gravity, according to GR is caused by the presence of matter and energy, which tells space how to curve, and that curvature tells matter how to move. Eventually, the scientific community will come to a consensus on gravity, and the work effort is toward quantum gravity, as I understand it.

Since it seems right to say that the quantum field occupies all space, and photons being electromagnetic radiation, would logically be traversing the field at the speed of light, their velocity is presumably governed by the local density of the vacuum.

What ever relative velocity that is, c is always the same in a perfect vacuum. However, we know that the energy density of the vacuum in QFT can vary, and is never a perfect vacuum, and so the velocity of light will vary from one level of vacuum energy density to another, and therefore from one bubble to another. Some might take exception to that, and so it is open to discussion.

In the mean time, for now I’ll take my musings about how I interpret the QFT Wikis, and cosmology in general, to my threads On the Lighter Side.

Title: Re: False Vacuum; Who, What, Where, When, Why?
Post by: jeffreyH on 11/08/2018 12:56:01
What if antimatter has a bias towards manifesting within a true vacuum. While matter has the opposite and tends to favour a false vacuum with a particular energy. This will allow two co-existing vacuum states. Virtual particle pairs can manifest temporarily in either domain. The true vacuum will have far more antimatter than matter. If the false vacuum collapses then all matter and antimatter will meet and annihilate. This is equivalent to nothing, except the true vacuum of course. This then leaves the connundrum of where the false vacuum came from.
Title: Re: False Vacuum; Who, What, Where, When, Why?
Post by: Bogie_smiles on 11/08/2018 20:32:17
What if antimatter has a bias towards manifesting within a true vacuum. While matter has the opposite and tends to favour a false vacuum with a particular energy. This will allow two co-existing vacuum states. Virtual particle pairs can manifest temporarily in either domain. The true vacuum will have far more antimatter than matter. If the false vacuum collapses then all matter and antimatter will meet and annihilate. This is equivalent to nothing, except the true vacuum of course. This then leaves the connundrum of where the false vacuum came from.
I'm not sure anyone else will take a shot at it, lol, so here is a link from years back (2010) that refers to experiments from the 1990s, but touches on your topic. The quote I pulled from the article is a hint to its content, but viewers should read the article do their own research to get the perspective:
https://phys.org/news/2010-12-theoretical-physics-breakthrough-antimatter-vacuum.html (https://phys.org/news/2010-12-theoretical-physics-breakthrough-antimatter-vacuum.html)

Quote from: phys.org
At the heart of this work is the idea that a vacuum is not exactly nothing.
"It is better to say, following theoretical physicist (https://phys.org/tags/theoretical+physicist/) Paul Dirac (https://phys.org/tags/paul+dirac/), that a vacuum, or nothing, is the combination of matter and antimatter -- particles and antiparticles. Their density is tremendous, but we cannot perceive any of them because their observable effects entirely cancel each other out," Sokolov said.
You could be talking about how the observed matter in the universe can exist if matter and antimatter annihilate each other, and are concerned about the conundrum of first cause, if the "always existed" solution isn't invoked. If the way that the existing matter came into existence was spontaneous symmetry breaking, your proposal would be aimed at explaining why any matter exists after the annihilation accompanying the initial symmetry breaking event, right? I haven’t fully researched the concept of symmetry breaking in regard to the false or true vacuums of QFT, but I know there is a lot of material along that line that we can get into if it isn't too speculative for this sub-forum.
Title: Re: False Vacuum; Who, What, Where, When, Why?
Post by: yor_on on 13/08/2018 01:51:19
This is the main stream definition

https://cosmosmagazine.com/physics/vacuum-decay-ultimate-catastrophe
Title: Re: False Vacuum; Who, What, Where, When, Why?
Post by: Bogie_smiles on 13/08/2018 02:27:52

This is the main stream definition


https://cosmosmagazine.com/physics/vacuum-decay-ultimate-catastrophe (https://cosmosmagazine.com/physics/vacuum-decay-ultimate-catastrophe)
Thanks for the link. That is an enjoyable read, and right on topic, and I’m posting the text becasue I am going to quote from it in my next reply:

Vacuum decay: the ultimate catastrophe
Of all the ways the Universe might die, vacuum decay is the most efficient.

Conceptual illustration of the Higgs Field that physicists believe permeates the Universe, and that could theoretically bring about its end. – DAVID PARKER / Getty Images [image deleted]
Every once in a while, physicists come up with a new way to destroy the Universe. There’s the Big Rip (a rending of spacetime), the Heat Death (expansion to a cold and empty Universe), and the Big Crunch (the reversal of cosmic expansion). My favourite, though, has always been vacuum decay. It’s a quick, clean and efficient way of wiping out the Universe.
To understand vacuum decay, you need to consider the Higgs field that permeates our Universe. Like an electric field, the Higgs field varies in strength, based on its potential. Think of the potential as a track on which a ball is rolling. The higher it is on the track, the more energy the ball has.
The Higgs potential determines whether the Universe is in one of two states: a true vacuum, or a false vacuum. A true vacuum is the stable, lowest-energy state, like sitting still on a valley floor. A false vacuum is like being nestled in a divot in the valley wall – a little push could easily send you tumbling. A universe in a false vacuum state is called “metastable”, because it’s not actively decaying (rolling), but it’s not exactly stable either.
There are two problems with living in a metastable universe. One is that if you create a high enough energy event, you can, in theory, push a tiny region of the universe from the false vacuum into the true vacuum, creating a bubble of true vacuum that will then expand in all directions at the speed of light. Such a bubble would be lethal.
“VACUUM DECAY IS THE ULTIMATE ECOLOGICAL CATASTROPHE ... NOT ONLY IS LIFE AS WE KNOW IT IMPOSSIBLE, SO IS CHEMISTRY ...”
The other problem is that quantum mechanics says that a particle can ‘tunnel’ through a barrier between one region and another, and this also applies to the vacuum state. So a universe that is sitting quite happily in the false vacuum could, via random quantum fluctuations, suddenly find part of itself in the true vacuum, causing disaster.
The possibility of vacuum decay has come up a lot lately because measurements of the mass of the Higgs boson seem to indicate the vacuum is metastable. But there are good reasons to think some new physics will intervene and save the day.
One reason is that the hypothesised inflationary epoch in the early Universe, when the Universe expanded rapidly in the first tiny fraction of a second, probably produced energies high enough to push the vacuum over the edge into the true vacuum. The fact that we’re still here indicates one of three things. Inflation occurred at energies too low to tip us over the edge, inflation did not take place at all, or the Universe is more stable than the calculations suggest.
If the Universe is indeed metastable, then, technically, the transition could occur through quantum processes at any time. But it probably won’t – the lifetime of a metastable universe is predicted to be much longer than the current age of the Universe.
So we don’t need to worry. But what would happen if the vacuum did decay?
The walls of the true vacuum bubble would expand in all directions at the speed of light. You wouldn’t see it coming. The walls can contain a huge amount of energy, so you might be incinerated as the bubble wall ploughed through you. Different vacuum states have different constants of nature, so the basic structure of matter might also be disastrously altered. But it could be even worse: in 1980, theoretical physicists Sidney Coleman and Frank De Luccia calculated for the first time that any bubble of true vacuum would immediately suffer total gravitational collapse.
They say: “This is disheartening. The possibility that we are living in a false vacuum has never been a cheering one to contemplate. Vacuum decay is the ultimate ecological catastrophe; in a new vacuum there are new constants of nature; after vacuum decay, not only is life as we know it impossible, so is chemistry as we know it.
“However, one could always draw stoic comfort from the possibility that perhaps in the course of time the new vacuum would sustain, if not life as we know it, at least some creatures capable of knowing joy. This possibility has now been eliminated.”
To know for sure what would happen inside a bubble of true vacuum, we’d need a theory that describes our larger multiverse, and we don’t have that yet. But suffice it to say, it would not be good. Luckily, we’re probably reasonably safe.
At least for now.
Title: Re: False Vacuum; Who, What, Where, When, Why?
Post by: Bogie_smiles on 14/08/2018 23:51:42
In response to the link from yor-on, first let me say that I like the perspective from which David Parker writes the article “Vacuum decay: the ultimate catastrophe”. The perspective being, that of all the ways the Universe might die, vacuum decay is the most efficient. It makes for good and exciting reading, and it lets the mind wander if it wants, but let’s be serious. The article talks of the universe as if it was a finite entity upon which universe-wide events could befall the whole thing, and maybe even in an instant.
 


To have that perspective, you are dealing with events worthy of qualifying as “first cause” events, like the universe beginning from spontaneous symmetry breaking where matter and antimatter separate out of a seeming nothingness, sparked by some random perturbation. At the very least such a preexisting “nothingness” must have had the potential for such a random event, and therefore it wasn’t “nothing” to begin with (think always existed). Given that reasoning, such random spontaneous events, and their ultimate catastrophic (universal) “deaths” should at least be considered repetitive. That means the cataclysmic, catastrophic universal death wasn’t all that bad, because in the next trillion years or so things are back and ready for another ultimate catastrophe.

But that is not in character with the nature of QFT according the Wiki sources at which we have been looking. For example, it seems perfectly consistent with QFT that there is just one universe, and no reason to believe that it isn’t infinite and eternal; a concept that rests well with logic. How can any single event have the potential to swept the entire universe clean of the potential for life, forever? It isn’t going to happen, simply because such universe-wide events can’t occur everywhere at once due to distance and time constraints; “infinite and eternal” trumps any such coordination occurring around a single event.

Cataclysms would logically befall all of the living inhabitants and all of the particulate matter associated with every major bubble collision, and with the inflationary expansion associated with such collisions. But I am a practical sort, and I don’t see suggestions as to how even the most cataclysmic peculiarities that might befall an infinite and eternal universe could spell complete and utter doom that goes beyond an isolated event among the local nucleating bubbles.

Don’t get me wrong, events like big bangs, and collisions of nucleating bubble walls would utterly destroy life within the reach of such an event, if only long enough for things to reshape themselves. But still they would qualify as local events. They don’t spike any concern on my part for the continuation of the greater universe, and its ability to host habitable environments, where life can be generated and evolved through iterations, again and again, here and there.


 It brings out the poet in me:

Explosions then, great cataclysms, Booms, it’s an inferno.
Bubbles crash and burst around, as if they weren’t eternal.

But when the dust clouds settle in and particles iterate,
Then out come living molecules; it’s nucleated fate.

Reply to yor_on's link to be continued ...
Title: Re: False Vacuum; Who, What, Where, When, Why?
Post by: Bogie_smiles on 15/08/2018 13:33:00
My last post, notwithstanding the poem, addresses what could be considered a myth that is often featured in theoretical cosmological models, and that is that the fate of the entire universe can turn on a single event.

One of the features I like about QFT is random bubble nucleation, and how nicely it can be equated with what we know and see in our own visible universe. It strongly hints that, in the context of QFT, we are inside of one of a potentially infinite number of nucleated bubbles.

My layman takeaway from the Wikis (open for comment and discussion), and from the @yor_on link to the Cosmos Magazine article, is that bubbles can occur randomly across potentially infinite space and time. Though the occurrence can be described as random in regard to where and when they occur, there is a continual process of new bubble nucleation. I would expect that there is more than pure mathematical randomness to the process because a nucleated bubble that actually produces a version or replica of what we observe around us would be preceded by tunneling and/or bubble collision. Though the “where and when” of collision events might not be predictable because of a lack of data, if in fact we had an overview of a large enough patch of the vacuum, and could measure density data to plot out the vacuum energy density of the surrounding bubbles, my expectation is that we could identify a set of natural preconditions to tunneling, bubble collisions, and nucleation of new bubbles like ours. 

The reference to “tunneling” is a way to acknowledge that the bubble “playing field” features close proximity and interaction between the walls of nucleating bubbles, and given some set of conditions, false vacuums can achieve true vacuum status. The low vacuum energy density of the false vacuum can be boosted to a higher vacuum energy density, up to or somewhere closer to the vacuum density of the true vacuum.

This is referred to as the collapse of the false vacuum, and is referred to as a catastrophic event. A result of the potentially extreme density of the new bubble would negate existing particles, void the effective physics and chemistry going on there, and set the stage for a new bubble to establish its own operative physics and chemistry. However, there is nothing to say that the natural set of preconditions wouldn’t lead to the reestablishment of the same laws, resulting in the same physics and chemistry in the new bubble.


If there is a repetitive process that does all of that, then the metastability of the false vacuum decay, as mentioned in the QFT Wikis and articles is of note. Does it mean that what seems to be the resulting catastrophe that false vacuum decay would represent to the occupants and contents of the affected false vacuum bubble, might, like the article hints, give us “good reasons to think some new physics will intervene and save the day”?

Title: Re: False Vacuum; Who, What, Where, When, Why?
Post by: jeffreyH on 15/08/2018 14:32:09
An article of interest.
http://scholar.google.co.uk/scholar_url?url=https://arxiv.org/pdf/gr-qc/0503090&hl=en&sa=X&scisig=AAGBfm3pHMd7HXOHV_J1Or6sk-apZBGrWg&nossl=1&oi=scholarr
Title: Re: False Vacuum; Who, What, Where, When, Why?
Post by: Bogie_smiles on 15/08/2018 23:23:36
An article of interest.
http://scholar.google.co.uk/scholar_url?url=https://arxiv.org/pdf/gr-qc/0503090&hl=en&sa=X&scisig=AAGBfm3pHMd7HXOHV_J1Or6sk-apZBGrWg&nossl=1&oi=scholarr (http://scholar.google.co.uk/scholar_url?url=https://arxiv.org/pdf/gr-qc/0503090&hl=en&sa=X&scisig=AAGBfm3pHMd7HXOHV_J1Or6sk-apZBGrWg&nossl=1&oi=scholarr)

Yes, indeed.

Abstract
Our Universe may be a domain separated by physical phase boundaries from other domain-Universes with different vacuum energy density and matter content. The coexistence of different quantum vacua is perhaps regulated by the exchange of global fermionic charges or by fermion zero modes on the phase boundary. An example would be a static de-Sitter Universe embedded in an asymptotically flat spacetime.
——————
Thanks and I’ll have to work on that one a bit :) before I start talking about it.


However, it does contribute to the idea that QFT is a type of madness, as suggested in this very timely video:


The way that I put it all into perspective is to try to grow my learning at a faster rate than I lose my mind; net gain and all that, lol.
Title: Re: False Vacuum; Who, What, Where, When, Why?
Post by: Bogie_smiles on 21/08/2018 13:57:21
Reply #69

I think the JeffreyH link is a topic of interest to a much more sophisticated audience than layman enthusiasts like me. The reason I struggle with it is probably just that I am not well enough informed to appreciate the rigor behind the paper’s perspective. It focuses on a connection between bubbles and bubble walls that can be separated by domain boundaries, and where emergent gravity in different vacuum domains can have a cause other than the notion of spacetime. It speculates that the boundary separation is regulated by a difference in the boundary charges referred to as fermionic charges or fermion zero modes on the phase boundary.

I do like the way it addresses coordinate singularities that can arise under those circumstances. It supports the concept that domain-Universes can/will have different vacuum densities and matter content, and while they remain separate, might be governed by different physics that yield different intensities of the local forces.

Being a layman and limited by layman level thinking, the paper is consistent with the position that there is some grand order to the greater Universe. The grand order would be displayed by a common larger set of invariant natural laws. In that case there is logic that the resulting lesser local set of laws that come into play at the low end of the density range, as separate domain bubbles expand, can disappear (work themselves out) via chaotic events like big bangs when expanding bubbles collide. Big bangs bring the higher vacuum energy densities to the nucleated bubble domains and therefore would unite the forces, making for a fresh start within the new domain.

Such massive events could naturally break down the domain boundaries when simple spatial overlaps occur due to bubble expansion. That might mean that the curvature of spacetime corresponds to the local domain-Universe that emerges after separated domain-universes collide and produce big bangs.

During the chaos of those collisions, Einstein’s solution to gravity might be put on hold until the collision sorts itself out. A sort of emergent orderliness within domains that are otherwise separated by corridors of time and space where a broader set of natural laws are in place. Those periods of chaos could create energy density environments that have more parameters; parameters that supersede the conditions that allow for an orderly space time gravity. The paper seems to invoke the emergence of stable domains with quiet internal places referred to as separate domain-Universes where Einstein’s solution of curved spacetime comes back into vogue.

Database Error

Please try again. If you come back to this error screen, report the error to an administrator.
Back